LSAT and Law School Admissions Forum

Get expert LSAT preparation and law school admissions advice from PowerScore Test Preparation.

 Administrator
PowerScore Staff
  • PowerScore Staff
  • Posts: 8919
  • Joined: Feb 02, 2011
|
#36390
Complete Question Explanation

Weaken—CE. The correct answer choice is (D)

The stimulus uses a variation of the classic “Some people claim” argument structure. In this
case, computer manufacturers and retailers claim that the complexity involved in connecting the
components of personal computers is not a barrier to their use. As is usual with this structure, the
author denies that claim, concluding that “this is wrong.” The remainder of the stimulus is comprised
of two premises that provide support for the author’s conclusion and show the causes of the complexity.

You are asked to weaken the argument, which is benefi cial since most students feel the argument has
several holes. If you felt that way, use your gut reaction to prephrase possible answers. If you did not
feel the argument had a hole before seeing the Weaken question, ask yourself if the premises truly
prove the author’s conclusion, and what might counter the author’s argument.

Answer choice (A): This response attracted many test takers, since they assumed that it would make
it less likely that the addition of accessories was diffi cult. However, explaining the purpose of a
switch or jumper does not necessarily clarify the proper settings, and this response does not address
the problems with accompanying software. Most importantly, this answer does not attack the idea in
the stimulus that customers “have to take full responsibility for the settings of jumpers and switches.”
This answer does not change the fact that customers still have to deal with the settings.

Overall, this was by far the most popular incorrect answer.

Answer choice (B): The cost of the software for accessories has nothing to do with the issue of
diffi culty of proper installation, so this response is incorrect.

Answer choice (C): Since the argument concerns the present, this response, which concerns the
future, is off-topic and incorrect.

Answer choice (D): This is the correct answer choice. If personal computers are sold as a
package including accessories and free installation, the diffi culties described in the stimulus become
irrelevant as an expert, not the consumer, will resolve the problems. In the case of installation, cost
is relevant, because a costly installation might inspire many consumers to do it themselves, thus
making the claims about diffi culty more compelling. In this response, the suggestion that installation
is free counters the idea that consumers would need, or want, to install components on their own.

Answer choice (E): This response serves to strengthen the argument by showing that manufacturers
do not knowingly or intentionally try to make it easier to install accessories.

If you selected this choice, you may have misread the stimulus and incorrectly identifi ed the
manufacturer’s position as the conclusion, or misunderstood the question as asking you to help the
author attack the manufacturers.
 Echx73
  • Posts: 36
  • Joined: Nov 11, 2015
|
#23348
Hi Team,

I have a question, I was wondering if you could give some direction on why answer D is better than A (The correct answer is D). I was reading your LR Bible and they say with weakening questions "most" of the time the correct answer will weaken the conclusion. I feel answer A attacks the conclusion more than D.

My other question pertaining to my first is, since 'most' means a majority but not necessarily all, then the rare times when we are not suppose to attack the conclusion but we are suppose to attack the premise(s)? If so, how do I distinguish between attacking the conclusion and attacking the premise?

Eric
 David Boyle
PowerScore Staff
  • PowerScore Staff
  • Posts: 836
  • Joined: Jun 07, 2013
|
#23361
Echx73 wrote:Hi Team,

I have a question, I was wondering if you could give some direction on why answer D is better than A (The correct answer is D). I was reading your LR Bible and they say with weakening questions "most" of the time the correct answer will weaken the conclusion. I feel answer A attacks the conclusion more than D.

My other question pertaining to my first is, since 'most' means a majority but not necessarily all, then the rare times when we are not suppose to attack the conclusion but we are suppose to attack the premise(s)? If so, how do I distinguish between attacking the conclusion and attacking the premise?

Eric

Hello Eric,

Answer A is not very helpful, since even if the manuals explain, that doesn't mean that the average customer understands them. Also, the manuals might not explain the interaction of various jumpers and switches, which it might take a professional installer to understand easily.
And, attacking the premise is fairly rare, since, among other things, it might involve contradicting something that was already said. But if that happens, feel free to try attacking the premise. E.g., the stimulus says, "Joey claims that it is raining outside. Therefore it is unsafe to go outside." Normally, an good answer might attack the conclusion by saying something like, "But everyone has umbrellas and rain boots." This would show that maybe it isn't unsafe to go outside. But it's possible an answer could say, "Joey is a big fat liar and it isn't really raining outside", which would attack the premise.

Hope this helps,
David
 KG!
  • Posts: 69
  • Joined: May 26, 2020
|
#77258
Hi,

I got this question correct, but I noticed a problem with getting answers correct is that I sometimes don't see what is wrong with the argument on my initial read.

I was wondering if your were able to walk through my thought process/ help me clearly see the flaws esp if its a common flaw.
My thought process (esp because of the way the conclusion was worded --- they already did the work for me)
1. Find the answer choice that makes it likely that computer manufactures and realtors tell us that the complexity involved in connecting the various components of personal computers IS NOT a widespread obstacle...."

To find the flaw (although that wan't our purpose in this question) I find that
1. Well we are not sure if customers ALWAYS have to take full responsibility. To put in LSAT terms, the author overlooks the possibility that some customers may pay an extra service to have the accessories added.
2. Just because MANY accessories require extra software isn't enough to mean that its a huge obstacle for customers.

Are my assessment of the flaws enough and correct?
User avatar
 KelseyWoods
PowerScore Staff
  • PowerScore Staff
  • Posts: 1079
  • Joined: Jun 26, 2013
|
#77476
Hi KG!

I think you prephrased how to weaken this argument well: the conclusion is that the complexity involved in connecting the various components of personal computers is a widespread obstacle use to their use so to weaken it we should find an answer choice that shows that it is NOT a widespread obstacle.

I think your identification of the flaws in this argument are also on the right track. When identifying flaws, identify the conclusion, identify the premises, and then think about how the premises support the conclusion and why they don't fully support the conclusion.

So this argument is basically:

P: The complexity involved in connecting components to PCs is an obstacle for customers who install accessories
P: Many accessories require extra software that can interfere with other accessories
C: The complexity involved in connecting components to PCs is a widespread obstacle

As you noted, there are a couple of problems here. First, we don't know how many customers actually install accessories themselves. If only a small number of customers do this, then it is not a widespread obstacle. Second, "many" is a very vague term that basically has no specific meaning on the LSAT. And it just says that the extra software required by these "many" accessories can cause other accessories to stop working but doesn't say how often this actually occurs. So the last sentence also does nothing to tell us how widespread of an issue this is.

Good job looking for those flaws and prephrasing this Weaken question!

Hope this helps!

Best,
Kelsey

Get the most out of your LSAT Prep Plus subscription.

Analyze and track your performance with our Testing and Analytics Package.